Difference between revisions of "2016 AMC 10A Problems/Problem 14"

(Solution)
(Solution)
Line 3: Line 3:
 
The amount of twos in our sum ranges from <math>0</math> to <math>1008</math>, with differences of <math>3</math> because <math>2 \cdot 3 = lcm(2, 3)</math>.
 
The amount of twos in our sum ranges from <math>0</math> to <math>1008</math>, with differences of <math>3</math> because <math>2 \cdot 3 = lcm(2, 3)</math>.
  
The possible amount of twos is <math>\frac{1008 - 0}{2} + 1 \Rightarrow \boxed{\textbf{(B)} 337</math>
+
The possible amount of twos is <math>\frac{1008 - 0}{2} + 1 \Rightarrow \boxed{\textbf{(C)} 337}</math>

Revision as of 18:59, 3 February 2016

Solution

The amount of twos in our sum ranges from $0$ to $1008$, with differences of $3$ because $2 \cdot 3 = lcm(2, 3)$.

The possible amount of twos is $\frac{1008 - 0}{2} + 1 \Rightarrow \boxed{\textbf{(C)} 337}$